1. Trang chủ
  2. » Giáo Dục - Đào Tạo

Báo cáo học viên nghệ an

165 149 0

Đang tải... (xem toàn văn)

Tài liệu hạn chế xem trước, để xem đầy đủ mời bạn chọn Tải xuống

THÔNG TIN TÀI LIỆU

Thông tin cơ bản

Định dạng
Số trang 165
Dung lượng 5,49 MB

Nội dung

ỨNG DỤNG SỐ PHỨC VÀO ĐA THỨC Trần Văn Châu ( Trường THPT Chuyên LÊ THÁNH TÔNG Thành phố Hội An Quảng Nam) Tromg kỳ thi học sinh giỏi thường xuất toán Đa thức Bài viết xin trình bày phương pháp dùng số phức để giải số dạng tập Đa thức 1.LÝ THUYẾT CĂN BẢN VỀ SỐ PHỨC a/ Dạng đại số : z = a+ib với a;b số thực i2=-1 Số phức liên hợp: z  a  ib b/ Dạng lượng giác : z  r (cos   i sin  )  acgumen z r mô đun z c/Dạng mũ: z  rei d/Số phức z gọi bậc n đơn vị thỏa mãn đẳng thức z n   Các bậc n đơn vị : cos k k  i sin ; k  0;1; n  n n Nếu z  bậc n đơn vị ; suy z nghiệm phương trình z n 1  z n    z   Như phương trình z n 1  z n    z   có n-1 nghiệm cos k k  i sin (k  1; 2; ; n  1) n n e/Công thức Moive : z  r (cos   i sin  ) Khi z n  r n (cos n  i sin n ) CHIA HẾT TRONG ĐA THỨC Bài tập : Tìm n để đa thức P( x)  x n  x n 1 Chia hết cho đa thức Q( x)  x  x  Lời giải : Đa thức Q(x) có nghiệm   cos 2 2  i sin 3 Đa thức P(x) chia hết cho đa thức Q(x) P ( )  Áp dụng cơng thức Moive ta có cos  cos 4n 4n 2n 2n  i sin  cos  i sin 1  3 3 2n 2n 2n 2n 2n (2 cos  1)  i sin (2 cos  1)   cos 1  3 3  n  3k  1; n  3k  Bài tập Cho dãy đa thức xác định fo ( x)  2; f1 ( x)  3x; f n ( x)  3xf n1 ( x)  (1  x  x2 ) f n2 ( x); n  Tìm tất giá trị n để đa thức f n ( x) chia hết cho đa thức g ( x)  x3  x  x Lời giải: Ta tìm số hạng tổng quát dãy đa thức cách xét dãy số sau u0  2; u1  3x; un 3x.un1  (1  x  2x2 ).un2 Xét phương trình đặc trưng t  3x.t  x  x    t  x  1; t  x  Suy un   ( x  1)n   (2x 1)n Dựa vào u0 ; u1 ta tìm     Do fn (x)  (x  1)n  (2x  1)n Do f n ( x) chia hết cho g(x) nên f (0)  f ( )  f ( )  Với  nghiệm phương trình x  x   Ta có f n (0)   n  2k  Khi f n ( )  (2 1)n  (  1)n  (2 1)(4  4  1)k  (  1)(  2  1)k  (2  1)(3)k  (  1)(3 )k  (3) k  (2 1)  (  1)( ) k  Do f n ( )  nên ta có (  1)( )k  2   Ta xét trường hợp sau +Với k=3t (  1)( )k  2   (  1)( )3t  2      2   3  (không thỏa) + Với k=3t+1 (  1)( )k  2   (  1)( )3t 1  2   (   )  2   (thỏa) +Với k=3t+2 (  1)()k  2   (  1)()3t 2  2      2   (không thỏa) Kết luận : n=6t+3 XÉT TÍNH KHẢ QUY CỦA ĐA THỨC Bài 1: Chứng minh khơng thể phân tích đa thức f ( x)  ( x  12 )(x  22 ) (x  n2 )  thành tích hai đa thức hệ số nguyên bậc lớn hay Lời giải : Giả sử ngược lại , ta có f(x)=g(x).h(x) với g(x) h(x) đa thức hệ số nguyên có bậc lớn hay Ta để ý f (ki)  f (ki)  1; k  0,1, , n ,ta có g (ki )h(ki )   g (ki )h( ki); k  1, 2, , n Vì số có bốn cách phân tích i  1.1 ; (-1).(-1) ;(i).(-i) ; (-i).(i) Nên ta có  g (ki); h(ki)  (1;1);(1; 1);( i;i);(i;  i) với k 1; 2; ;  n;1;2; ; n Như bốn trường hợp ta có g (ki)  h(ki)  h(ki) Như đa thức P(x) =g(x)-h(-x)có 2n nghiệm phân biệt bậc P(x) nhỏ 2n g(x)=h(-x) Suy degg(x)=degh(x)=n Vì f(x) đa thức monic nên g(x) h(x) monic Khi đa thức g ( x)  h ( x) có bậc nhỏ 2n Đa thức có 2n nghiệm ki với k=1;2; ;n;-1;-2; ;-n Suy g ( x)  h ( x) Do g(x) h(x) monic nên g(x)=-h(x) khơng xảy ta có g(x) =h(x) Khi f ( x)  g ( x) suy f (0)   g (0) 2  (n!)2  điều khơng thể g(0) số nguyên Bài 2: Cho đa thức P( x)  an xn  an1xn1   a1 x  a0( an  0; n 2) có hệ số số nguyên Chứng minh tồn vô số số nguyên k cho P(x)+k khơng phân tích thành tích hai đa thức hệ số nguyên với bậc lớn n Lời giải: Ta chọn k  p  a0 với p số nguyên tố thỏa mãn p   | | i 1 Vì có vơ số số ngun tố nên có vơ số k Ta chứng minh Q( x)  P( x)  k  a n xn   a1x  p bất khả quy  x  Trước hết ta nhận xét : Nếu  nghiệm phức Q(x)   Vậy   p  an n   a1  an   a1 mâu thuẩn với cách chọn p Giả sử Q(x) khả quy Tức Q(x)=g(x).h(x) với g(x), h(x) đa thức có hệ số số nguyên degg(x)>0 ;degh(x) >0 Ta có p=Q(0)=h(0).g(0) p ngun tố nên g(0)=1 hay h(0)=1 + Nếu g(0)=1và giả sử g(x)= bk x k   b1 x  (do g(0)=1, nên số hạng tự g(x) 1) Gọi z1 ; ; zk nghiệm phức đa thức g(x) suy chúng nghiệm đa thức Q(x) Theo nhận xét ta có z1.z2 zk  Mặt khác theo định lý Vietta lại có z1 zk  1 bk (mâu thuẩn) Vậy Q(x) bất khả quy Z[x] ỨNG DỤNG ĐỂ TÌM ĐA THỨC Bài tập 1:Tìm tất đa thức P(x) thỏa điều kiện : P( x) P( x  1)  P( x ) Lời giải: Giả sử  nghiệm phương trình P(x) = ta nhận thấy: +  ; ; ;  n; nghiệm +   1;  1; ; 2n  1; nghiệm Do   hay   có    1;    Giả sử      Ta đặt   cos   i sin      cos  1    ;  5  Ta xét   : Do   nghiệm P(x)=0    (cos 2 2  1)  sin 3 3 Tương tự   5 (vô lý) dẫn đến điều vô lý Như ta kết luận   0;   Từ suy P( x)  k.x m (1  x)n Thay vào phương trình ban đầu ta có k=1 m=n Ngồi đa thức P(x)=C P(x) = P(x) =1 Thử lại ta có kết luận : P(x)=0; P(x)=1; P( x)  x n (1  x)n Bài tập : Tìm tất đa thức P(x) khác đa thức cho P( x).P( x  1)  P( x  x  1), x  (1) Lời giải: Giả sử x0 nghiệm P(x)=0 x02  x0  nghiệm Thay x x-1 vào (1) ta có P( x  1).P( x)  P( x  x  1) nên x02  x0  nghiệm Giả sử  nghiệm có mơ đun lớn Từ cách chọn ta suy             Ta lại có        (    1)                 Dấu đẳng thức xảy nên      k (    1), k  Từ tính lớn  ta suy          |  | k  1 Do      (    1)        i Như P(x)  ( x 1) m.Q( x) , Q(x)khơng chia hết cho x  Thay ngược vào (1) ta có Q( x).Q(x  1)  Q(x  x  1)x  (2) Nếu Q(x)=0 lại có nghiệm lại có Q(x) phải chia hết cho x  (vô lý) Suy Q(x) =C Thay vào (2) có C=1 Vậy đa thức P( x)  ( x  1)m Thử lại kết luận P(x)  ( x 1) m 5.TÌM ĐA THỨC NHẬN SỐ PHỨC LÀM NGHIỆM Bài : Tìm đa thức có hệ số ngun bậc nhỏ nhận số   1 2i làm nghiệm Lời giải :Do    2i     2i  (  1)2  4    2i   Vậy đa thức cần tìm P( x)  x  x  Giả sử tồn đa thức Q(x)=a.x+b (a;b nguyên) nhận  làm nghiệm; dễ thấy điều vơ lý Bài Tìm đa thức hệ số ngun bậc nhỏ nhận   2(cos 2 2  i sin ) 5 làm nghiệm Lời giải : Đặt     cos 2 2 +i sin 5  nguyên thủy bậc đơn vị Suy  nghiệm phương trình x5   Vậy đa thức cần tìm P( x)  x5  32 Việc chứng minh bậc nhỏ suy từ tính chât bậc n đơn vị Đa thức P(x) thỏa mãn P( x)  x5  32 LUYỆN TẬP Bài Tìm tất số nguyên dương n cho đa thức x3n1  x 2n  chia hết cho đa thức x  x  Bài 2: Chứng minh  x  1 n 1  x n chia hết cho x  x  Bài (IMO 1993) Chứng minh với n nguyên dương lớn đa thức P( x)  x n  x n 1  phân tích thành tích hai đa thức có bậc không nhỏ hệ số nguyên Bài : (USAMO 1976) Cho P(x) ;Q(x) ; R(x) ;S(x) đa thức thỏa mãn P( x5 )  x.Q( x5 )  x R( x5 )  ( x  x3  x  x  1) S ( x) Chứng minh P(x) chia hết xho x-1 Bài : Tìm tất đa thức P(x) thỏa mãn P( x).P(3x )  P(3x3  x) Bài 6: Cho dãy đa thức  Pn ( x)  thỏa mãn điều kiện : P1 ( x)  2x; P2 ( x)  2( x2  1); Pn ( x)  2x.Pn1 ( x)  ( x 1).Pn2 ( x); n  ; n  Chứng minh rằngđiều kiện cần đủ để đa thức Pn ( x) chia hết cho đa thức Q( x)  x  , n  4k  2; k  TÀI LIỆU THAM KHẢO [1] Nguyễn Chánh Tú ,Mở rộng trường lý thuyết trường Galois [2] Nguyễn Văn Mậu Trần Nam Dũng ,Số phức áp dụng [3] Tạp chí Tốn học tuổi trẻ [4] Nguyễn Văn Khuê Lê Mậu Hải,Hàm biến phức Hội An,Ngày 18/6/2017 PHƯƠNG PHÁP ĐẾM SỐ LẦN XUẤT HIỆN CỦA MỖI PHẦN TỬ TRONG MỘT TẬP HỢP ĐỂ GIẢI BÀI TOÁN TỔ HỢP Tác giả: Đỗ Văn Đức – Ngô Thị Hoa Trường THPT Chuyên Lương Văn Tụy - Ninh Bình Trong kì thi tuyển sinh đại học, thi học sinh giỏi cấp tỉnh, cấp quốc gia, cấp khu vực quốc tế, … lần có tốn tổ hợp Đây loại tốn khó có nhiều phương pháp để giải quyết, kể đến phương pháp truy hồi, công thức bao hàm loại trừ, sử dụng số phức, Sau đưa phương pháp giải toán tổ hợp nhờ đếm số lần xuất phần tử tập hợp thơng qua ví dụ Ví dụ (Đề thi học sinh giỏi Ba Lan) Cho tập hợp 𝑴 có 𝒏 phần tử hai tập tùy ý 𝑨, 𝑩 𝑴, xét cặp (𝑨, 𝑩) ta tính số phần tử 𝑨 ∩ 𝑩 Chứng minh tổng tất số phần tử giao gồm hai tập 𝑴 𝒏 𝟒𝒏−𝟏 Bài giải Xét phần tử 𝑎 ∈ 𝑀 Khi đó, có 2𝑛 − 2𝑛−1 = 2𝑛−1 tập 𝑀 có chứa phần tử 𝑎 Suy ra, có (2𝑛−1 )2 = 4𝑛−1 cặp hai tập 𝑀 có chứa 𝑎 , hay nói cách khác 𝑎 thuộc giao cặp 4𝑛−1 cặp Vậy phần tử 𝑎 ∈ 𝑀 đếm 4𝑛−1 lần ⟹ ∑ |𝐴⋂𝐵| = 𝑛 4𝑛−1 𝐴⊂𝑀 𝐵⊂𝑀 Theo cách giải trên, ta giải toán tổng quát sau Bài toán: Cho 𝑀 tập có 𝑛 phần tử, 𝐸 tập tất (có thứ tự) (𝐴1 , 𝐴2 , … , 𝐴𝑘 ) 𝐴𝑖 ⊂ 𝑀, 𝑘 số tự nhiên cho Chứng minh rằng: 𝑘 ∑ |⋂ 𝐴𝑖 | = 𝑛 2𝑘(𝑛−1) (𝐴1 ,𝐴2 ,…,𝐴𝑘 )∈𝐸 𝑖=1 Ví dụ (Đề thi Tiệp Khắc) Hãy tìm số cặp (khơng kể thứ tự) tập không giao tập hợp 𝒏 phần tử Bài giải Với số tự nhiên 𝑡 (𝑡 = ̅̅̅̅̅ 1, 𝑛) ta tìm (𝐴𝑖 , 𝐴𝑗 ) thỏa mãn: 𝐴𝑖 ∩ 𝐴𝑗 = ∅ |𝐴𝑖 | + |𝐴𝑗 | = 𝑡 Gọi 𝑀 tập 𝑛 phần tử đề Ta có 𝐶𝑛𝑡 cách chọn 𝑡 phần tử 𝑀 Với cách chọn 𝑡 phần tử ta có 𝐶𝑡𝑘 cách chọn từ 𝑘 phần tử để làm thành tập 𝐴𝑖 (𝑡 − 𝑘 ) phần tử lại tập 𝐴𝑗 𝑡 𝐶𝑡𝑘 Số cặp (𝐴𝑖 , 𝐴𝑗 ) là: ∑ = 2𝑡−1 𝑘=0 Vậy, với 𝑡 có 𝐶𝑛𝑡 2𝑡−1 cặp Do có cặp (∅, ∅) nên số cặp cần tìm là: 𝑛 𝑛 𝑡=1 𝑡=0 3𝑛 + 𝑡 𝑡−1 𝑡 𝑡 ∑ 𝐶𝑛 + = [∑ 𝐶𝑛 − 1] + = 2 Ví dụ (Olympic Châu Á Thái Bình Dương lần 10 /1997) Gọi 𝑴 = {𝟏, 𝟐, … , 𝟏𝟗𝟗𝟖}, cho trước số tự nhiên 𝒌 , 𝑨𝒊 (𝒊 = 𝟏, 𝟐, … , 𝒌) tập 𝑴 Giả sử 𝑭 tập tất (𝑨𝟏 ; 𝑨𝟐 ; … ; 𝑨𝒌 ) Hãy tính: 𝑺= |𝑨𝟏 ∪ 𝑨𝟐 ∪ … ∪ 𝑨𝒌 | ∑ (𝑨𝟏 ,𝑨𝟐 ,…,𝑨𝒌) ∈𝑭 Bài giải Đặt 𝑛 = 1998 Với 𝑖 ∈ 𝑀, gọi 𝑆𝑘 (𝑖)là số (𝐴1 , 𝐴2 , … , 𝐴𝑘 ) ∈ 𝐹 thỏa mãn: 𝑘 𝑖 ∈ ⋃ 𝐴𝑗 𝑗=1 Do có 2𝑛 tập 𝑀 2𝑛−1 tập 𝑀\{𝑖} ⟹ 𝑆𝑘 (𝑖) = 2𝑛𝑘 − 2(𝑛−1)𝑘 = 2(𝑛−1)𝑘 (2𝑘 − 1) Do đó, 𝑆 = 𝑛(2𝑘 − 1)2(𝑛−1)𝑘 Theo cách giải ví dụ trên, ta đưa số toán sau BẬC CỦA MỘT ĐỈNH TRONG ĐỒ THỊ Võ Quang Vinh, trường Chuyên Nguyễn Tất Thành Kon Tum Email: vqvinh82@gmail.com; ĐT: 0935039007 Trong viết này, giới thiệu số kiến thức lý thuyết đồ thị lớp toán liên quan đến việc đánh giá bậc đỉnh đồ thị để giải I Một số kiến thức đồ thị 1.1 Định nghĩa đồ thị - Một đồ thị vô hướng cặp G  (V , E) V tập hữu hạn đỉnh E  V V tập hữu hạn cặp đỉnh (khơng có thứ tự) – gọi tập cạnh - Với A, B V , ( A, B)  E A, B gọi hai đỉnh kề Cạnh ( A, B) đỉnh A (hay đỉnh B ) gọi liên thuộc với - Tập đỉnh kề với A ký hiệu N  A - Một cạnh nối đỉnh với gọi khun - Đồ thị đơn đồ thị khơng có khun hai đỉnh có nhiều cạnh Nếu khơng có ghi đặc biệt phần lại viết ta ngầm định đồ thị đồ thị đơn 1.2 Đồ thị con, đồ thị cảm sinh, đồ thị bù Cho đồ thị G  (V , E) - Đồ thị G '  (V ', E ') thoả mãn V '  V ; E '  E gọi đồ thị G  (V , E ) - G '  (V ', E ') đồ thị G  (V , E) với A, B V ' ( A, B)  E '  ( A, B)  E G '  (V ', E ') gọi đồ thị cảm sinh đồ thị G  (V , E ) lên tập V ' Ký hiệu G[V'] - Đồ thị H  (V , VxV\ E) gọi đồ thị bù đồ thị G  (V , E) 1.3 Bậc đỉnh đồ thị Cho đồ thị G  (V , E) A V - Số đỉnh kề với A gọi bậc A G  (V , E) kí hiệu degG A | N ( A) | - Một đỉnh có bậc gọi đỉnh lập Định lí Cho đồ thị đơn vô hướng G  (V , E) i)  degG A  | E | AV ii) Số đỉnh có bậc lẻ ln số chẵn Định lí Cho đồ thị G  (V , E) V '  V Với A V ' ta có degG A  degG[V '] A  degG[ V \V {A}] A    Định lí Cho đồ thị đơn vơ hướng G  (V , E) Khi  ( A; B )E (deg( A)  deg( B))   (deg A) AV 1.4 Đường đi, chu trình, đồ thị liên thơng Cho đồ thị G  (V , E) A, B V - Một đường từ A đến B dãy cạnh ( Ai , Ai 1 ), i  0, , k 1 với A0  A; Ak  B - Số cạnh đường gọi độ dài đường Ta quan tâm đến đường có độ dài lớn không sử dụng cạnh lần hay chí khơng sử dụng đỉnh q lần - Nếu ta có đường từ A đến B ( A, B)  E ta gọi chu trình Định lí Nếu đồ có bậc đỉnh lớn đồ thị cđó chứa chu trình II Một số toán liên quan đến bậc đỉnh đồ thị Với ý tưởng sử dụng đồ thị để giải toán tổ hợp nên tốn trình có lời giải đơn giản hơn, hay lời giải trình bày Bài tốn 1: Cho điểm phân biệt nằm đường tròn, nối điểm đường màu đỏ màu xanh Chứng minh rằng: a) Tồn tam giác có cạnh màu b) Tồn hai tam giác có cạnh màu Phân tích lời giải Ta coi điểm cho đỉnh đồ thị Nếu điểm nối với đường đỏ ta gọi cạnh Nếu điểm nối với đường xanh ta coi khơng phải đỉnh kề Ta thu đồ thị G  (V , E) Yêu cầu toán đặt chứng minh tồn chu trình có độ dài tồn đỉnh mà đồ thị cảm sinh tập gồm điểm khơng chứa cạnh a) Lấy đỉnh A V Ta phân hoạch tập đỉnh lại đồ thị thành tập, tập N ( A)  {B V | (A, B)  E} M ( A)  {B V | (A, B)  E} Khi | N ( A) |  | M(A) | theo nguyên lý Dirichlet tồn tập có số lớn Trường hợp 1: degG A  lấy đỉnh A1; A2 ; A3  N ( A) + Nếu tồn ( Ai ; Aj )  E ( A; Ai ; Aj ) tập điểm cần tìm + Nếu ngược lại ( A1; A2 ; A3 ) tập điểm cần tìm Trường hợp 2: degG A  ta có | M ( A) | lấy B1; B2 ;B3  M ( A) + Nếu tồn (Bi ; B j )  E ( A; Bi ; B j ) tập điểm cần tìm + Nếu ngược lại ( B1; B2 ; B3 ) tập điểm cần tìm b) Ta xét trường hợp Trường hợp 1: G tồn đỉnh có bậc lớn 4, lý luận tương tự trường hợp câu a) ta suy có tam giác khác thoả điều kiện đề Trường hợp 2: G tồn đỉnh có bậc bé 4, lý luận tương tự trường hợp câu a) ta suy có tam giác khác thoả điều kiện đề Trường hợp 3: đỉnh G có bậc Theo câu a) tồn tam giác thoả điều kiện đề Giả sử ( A1; A2 ; A3 ) có cách cạnh thuộc E  Khi tồn tam giác khác có ba đỉnh tốn giải  Nếu ngược lại ta giả sử N ( A1 )  {A2 ;A3 ;A4}  A2 ; A3  N ( A4 ) Xét đỉnh A4 , ta có  A1  N ( A4 ) degA   tương tự trường hợp câu a) ta suy tồn tam giác Tam giác thoả điều kiện đề khác với tam giác ( A1; A2 ; A3 ) Chú ý: việc sử dụng đồ thị vào giải tốn làm phức tạp lời giải, nhiên ví dụ đơn giản giúp làm quen với đồ thị Câu a tốn giải nguyên lý Dirichlet, câu b sử dụng bất đẳng thức đánh giá số góc có cạnh màu để giải Bài toán 2: Cho bảng 2018x2018 ô vuông, tô màu tuỳ ý số ô vuông bảng Chứng minh ta tơ màu thêm khơng q 4035 lại cho số ô tô màu hàng cột số chẵn Phân tích lời giải Nhận xét số hàng cột số chẵn nên số ô tô màu chẵn số ô không tô màu chẵn Ta xây dựng đồ thị G  (V ; E) hàng cột đỉnh đỉnh, ô giao hàng cột khơng tơ màu ta có cạnh đồ thị Bài tốn đưa việc chứng minh bỏ không 4035 cạnh để thu đồ thị mà bậc đỉnh số chẵn Vì khơng có thơng tin bậc đỉnh nên để tạo thêm giả thiết ta thay 2018+2018 n chứng minh phương pháp quy nạp Trước hết, ta chứng minh bổ đề: Bổ đề: từ đồ thị chứa n đỉnh bỏ không n  cạnh để thu đồ thị có bậc đỉnh số chẵn Chứng minh Ta chứng minh phương pháp quy nạp theo số đỉnh n đồ thị Thật vậy, kết hiển nhiên n  Giả sử với n  k ta chứng minh n  k  Xét đồ thị Gk 1 có k  đỉnh Trường hợp 1: Gk 1 tồn đỉnh lập A ta bỏ đỉnh A thu đồ thị có k đỉnh, áp dụng giả thiết quy nạp suy kết luận toán Trường hợp 2: Gk 1 tồn đỉnh A thoả degG A  ta bỏ đỉnh A cạnh liên kết với thu đồ thị có k đỉnh, áp dụng giả thiết quy nạp suy kết luận toán k 1 Trường hợp 3: đỉnh Gk 1 có bậc lớn Khi đó, Gk 1 tồn chu trình Để ý bỏ chu trình khơng làm thay đổi tính chẵn lẻ bậc đỉnh đồ thị Vì số cạnh đồ thị hữu hạn nên chứa hữu hạn chu trình, bỏ tất chu trình ta thu đồ thị có k  đỉnh thoả điều kiện trường hợp Sau áp dụng giả thiết quy nạp bổ sung vào lại chu trình bỏ ta thu đồ thị thoả điều kiện đề Vậy bổ đề chứng minh Áp dụng bổ đề với n  4036 ta thu kết tốn Bài tốn 3: Trong hội nghị có 2018 người tham gia Chứng minh tìm hai người hội nghị cho họ có số chẵn (có thể 0) người quen chung hội nghị Phân tích lời giải Ta xây dựng đồ thị G  (V ; E) người tham gia hội nghị đỉnh, hai người quen biết ta có cạnh đồ thị Bài toán đưa việc chứng minh tồn hai đỉnh A; B ( A  B ) cho | N ( A)  N ( B) | số chẵn (có thể 0) Chú ý | V | 2018 Giả sử ngược lại Lấy đỉnh A V , với X  N ( A) ta có degG[N (A)] X lẻ | N ( A)  N ( X ) | lẻ Điều với X nên suy | N ( A) | chẵn Do degG A chẵn Vì A chọn ngẫu nhiên nên suy G(V ; E ) gồm đỉnh bậc chẵn Vẫn xét đỉnh A ban đầu , với X  N ( A) ta có degG X   degG[N ( A)] X  degG[M ( A){X }] X  degG[M ( A){X }] X  degG X   degG[N ( A)] X Suy degG[M ( A){X }] X chẵn Mặt khác, với Y  M ( A) ta có degG[N( A){Y}] Y | N ( A)  N (Y ) | lẻ Do degG[M ( A)] Y  degG Y  degG[N ( A){Y }] Y lẻ Nên | M (A) | lẻ Điều mâu thuẫn với | V || N ( A) |  | M ( A) | 1  2018 Từ ta có điều phải chứng minh Bài tốn 4: Cho G  (V , E) đồ thị đơn vô hướng Chứng minh phân hoạch tập đỉnh G  (V , E) thành hai tập V1; V2 cho đồ thị cảm sinh hai tập có đỉnh có bậc chẵn Phân tích lời giải Đây tốn mà ta khơng có thơng tin đồ thị nên để giải tốn ta sử dụng ngun lý quy nạp nhằm giúp có thêm giả thiết để giải toán Chứng minh quy nạp theo số đỉnh đồ thị n Khi n  hiển nhiên ta có kết tốn Giả sử tốn với đồ thị có it n đỉnh Xét đồ thị G  (V , E) có n đỉnh Nếu tất đỉnh đồ thị có bậc chẵn ta chọn V1  V ;V2   tốn giải Nếu có đỉnh A0 V có bậc lẻ Để ý | N ( A0 ) | lẻ Nhận xét sau tách đỉnh A, sử dụng giải thiết quy nạp với đồ thị gồm n-1 đỉnh ta phân hoạch tập đỉnh thành tập cho đồ thị cảm sinh có bậc chẵn Hai tập thu có tính chẵn lẻ giao với N(A) khác Do ta phải thêm A vào tập giao với N(A) chẵn để đảm bảo bậc A đồ thị cảm sinh chẵn Tuy nhiên, bậc đỉnh khác đồ thị cảm sinh có bậc lẻ để giải vấn đề ta phải sử dụng đồ thị bù trước áp dụng giả thiết quy nạp Ta định nghĩa đồ thị H  (V \{A0 }, E ') có tập đỉnh gồm n  đỉnh từ G  (V , E) sau: + A, B N ( A0 ) ta có ( A; B)  E '  (A;B)  E +Nếu A N ( A0 ) B N ( A0 ) ( A; B)  E '  (A;B)  E Khi đó, theo giả thiết quy nạp V \{A0 } phân hoạch thành hai tập U1;U cho đồ thị cảm sinh hai tập có đỉnh có bậc chẵn Không tổng quát ta giả sử | N ( A0 )  U1 | chẵn | N ( A0 )  U2 | lẻ Ta đặt V1  U1  { A0 } ta chứng minh hai tập thoả yêu cầu toán  V2  U Thật vậy, xét X V1 ta có + Nếu X  A0 degG[V ] X  degG[V ] A0 | N ( A0 ) U1 | chẵn + Nếu X  N ( A0 ) ta có degG[V ] X  deg H[V ] X chẵn 1 + Nếu X  N ( A0 ) ta có: degG[V1 ] X  degG[V1 \ N ( Ao )] X  degG[V1 N( Ao ){X}] X   deg H[V1 \ N ( Ao )] X  | N ( A0 )  U1 | 1  deg H[V1 N( Ao ){X}] X | N ( A0 )  U1 | 2 deg H[V1  N( Ao ){X}] X  deg H[V1 )] X số chẵn Tương tự xét X V2 ta có + Nếu X  N ( A0 ) ta có degG[V ] X  deg H[V ] X chẵn + Nếu X  N ( A0 ) ta có: 1 degG[V 2] X  degG[V12 N ( Ao )] X  degG[V2 N( Ao ){X}] X  deg H[V1 \ N ( Ao )] X  | N ( A0 )  U1 | 1  deg H[V1 N( Ao ){X}] X | N ( A0 )  U | 1  deg H[V2 N( Ao ){X}] X  deg H[V2 )] X số chẵn Kết toán cho ta cách phân hoạch đỉnh đồ thị cho đỉnh đồ thị cảm sinh có bậc chẵn Tuy nhiên ta phân hoạch cho tính chẵn lẻ chúng khác Bài toán 5: Cho G  (V , E) đồ thị đơn vơ hướng Chứng minh phân hoạch tập đỉnh G  (V , E) thành hai tập V1; V2 cho: + Các đỉnh đồ thị G[V ] có bậc chẵn + Các đỉnh G[V 2] có bậc lẻ Phân tích lời giải Để thay đổi tính chẵn lẻ bậc đỉnh ta cần bỏ bớt đỉnh có cạnh nối với chúng Do nghĩ tới việc thêm vào đỉnh nối với tất đỉnh lại sau bỏ Lấy điểm O ngồi đồ thị, nối với tất đỉnh đồ thị ta thu đồ thị T Áp dụng kết toán số ta phân hoạch tập đỉnh T thành hai tập W1; W2 cho đồ thị cảm sinh hai tập có đỉnh có bậc chẵn Giả sử O  W2 , đặt V1  W1; V2  W2 \{O} , V1;V2 thoả mãn điều kiện đề Bài toán 6: Giả sử, nút giao thơng thành phố tổ hợp có bóng đèn cơng tắc Ấn vào cơng tắc thay đổi trạng thái bóng đèn nút bóng đèn nút kề (hai nút gọi kề có đường từ nút đến nút kia) Ban đầu tất bóng đèn tắt, chứng minh người ta bật sáng tất bóng đèn Phân tích lời giải Gọi S tập công tắc ta cần thay đổi trạng thái (1 lần) Ta coi mạng lưới đồ thị, ta cần chọn S cho đỉnh S nối với số chẵn lần đỉnh khác S , đỉnh không thuộc S nối với số lẻ lần đỉnh thuộc S Ta xây dựng đồ thị G  (V ; E) nút giao thơng đỉnh, hai nút giao thơng kề ta có cạnh đồ thị Lấy điểm O ngồi đồ thị, nối với tất đỉnh có bậc chẵn đồ thị G ta thu đồ thị H Theo toán ta phân hoạch tập đỉnh H thành V1; V2 cho đồ thị cảm sinh hai tập có đỉnh có bậc chẵn Khơng tổng quát giả sử O V1 , ta chọn S  V2 Ta có: + Nếu X  S degG[S ] X  degH[S ] X chẵn hay X nối với chẵn đỉnh thuộc S + Nếu X  S {O} ta xét trường hợp: i) X nối với O V1 degG X ;deg H [V ] X chẵn nên degG[S {X }] X  degG X   deg H (V ) X lẻ ii) X không nối với O V1 degG X lẻ deg H [V ] X chẵn nên degG[S {X }] X  degG X  deg H (V ) X lẻ Khi ta cần bậc tất cơng tắc thuộc S lần tốn giải 1 1 Bài toán 7: (China TST 1987) Cho số nguyên dương n (n  2) , mặt phẳng cho 2n điểm, nối chúng n2  đoạn thẳng Chứng minh rằng: a) Tồn tam giác b) Tồn hai tam giác có chung cạnh c) Tồn n tam giác Phân tích lời giải Ta xây dựng đồ thị Gn  (V ; E ) điểm đỉnh đỉnh, ô giao hàng cột không tô màu hai điểm nối với ta có cạnh đồ thị Ta chứng minh quy nạp theo n Khi n  ta có đồ thị gồm đỉnh cạnh Có thể thu từ đồ thị đầy đủ gồm đỉnh cách bỏ cạnh nên chứa tam giác có chung cạnh Hay kết luận a); b); c) toán a) Giả sử kết luận a) với n  hay Gn1 chứa tam giác Xét đồ thị Gn  (V ; E ) , lấy hai đỉnh A; B V cho ( A; B)  E Nếu tồn C  N ( A)  N ( B) ta có tam giác ( A; B; C ) Nếu N ( A)  N ( B)   , degG A 1  deg B B 1  2n  | VH | n  Xét đồ thị H  (VH ; EH )  G[V \{A;B}] có  2 | EH | n   (2 n   1)  (n  1)  10 đó, theo giả thiết quy nạp H chứa tam giác Từ suy Gn  (V ; E ) ln chứa tam giác b) Giả sử kết luận b) với n  hay Gn1 chứa tam giác có chung cạnh Xét đồ thị Gn  (V ; E ) , theo kết a) tồn tam giác Ta giả sử Gn  (V ; E ) chứa tam giác ( A; B; C ) degG A  deg B B  degG C Nếu có D  ( N ( A)  N ( B))  ( N (B)  N (C))  ( N (C)  N (A)) đỉnh ( A; B; C; D) tạo thành tam giác tam giác có chung cạnh Nếu ( N ( A)  N ( B))  ( N (B)  N (C))  ( N (C)  N (A))   degG A   deg B B   degG C   2n  suy degG A   deg B B   n Xét đồ thị H  (VH ; EH )  G[V \{A;B}] (bỏ đỉnh A, B) ta có | VH | n   theo giả thiết quy nạp ta suy H chứa hai tam giác có  | EH | n   n  (n  1)    chung cạnh Từ suy Gn  (V ; E ) ln chứa hai tam giác có chung cạnh c) Giả sử kết luận c) với n  hay Gn1 chứa tam giác có chung cạnh Xét đồ thị Gn  (V ; E ) , theo kết a) tồn tam giác Ta giả sử Gn  (V ; E ) chứa tam giác ( A; B; C ) degG A  deg B B  degG C Gọi t số tam giác Gn  (V ; E ) Theo nguyên lý bù trừ ta có | N ( A)  N ( B) |  | N (B)  N (C) |  | N (C)  N (A) | | N(A) |  | N(B) |  | N(C) |  | N(A)  N(B)  N(C) |  | N(A)  N(B)  N(C) | | N(A) |  | N(B) |  | N(C) | 2 n Do 11 t | N ( A)  N ( B) |  | N (B)  N (C) |  | N (C)  N (A) | 2 | N(A) |  | N(B) |  | N(C) | 2n Nếu | N(A) |  | N(B) |  | N(C) | 2n   n | N(A) |  | N(B) |  | N(C) | 3n  ta có kết tốn Nếu | N(A) |  | N(B) |  | N(C) | 2n   n | N(A) |  | N(B) |  | N(C) | 3n  2 Suy | N(A) |  | N(B) | (3n  2) Xét đồ thị H  (VH ; EH )  G[V \{A;B}] (bỏ đỉnh A, B) ta có | VH | n   theo giả thiết quy nạp ta suy H chứa n  tam  | EH | n   (3n  2)   (n  1)    giác Từ suy Gn  (V ; E ) ln chứa n tam giác Kết a) toán cho ta đánh giá tương quan số cạnh đồ thị với số đỉnh trường hợp đồ thị có chứa tam giác Tuy nhiên đánh giá chưa chặt kết cuối đồ thị có chứa n tam giác Định lí sau cho ta kết tổng quát trường hợp đồ thị không chứa tam giác Bài tốn 8: (Định lí Mantel – Turan) Xét đồ thị đơn vô hướng G  (V ; E) có  n2  n đỉnh k cạnh Khi đó, G  (V ; E ) khơng chứa tam giác k    4 Phân tích lời giải Chứng minh quy nạp theo số đỉnh đồ thị n Khi n  1, hiển nhiên ta có kết tốn Giả sử tốn với đồ thị có it n đỉnh Xét đồ thị G  (V , E) có n đỉnh khơng chứa tam giác Lấy cho ( A; B)  E ta có suy A; B V N ( A)  N ( B)   , degG A   deg B B   n  Gọi H  (VH ; EH )  G[V \{A;B}] gồm n  đỉnh Khi đó, H khơng chứa tam giác nên theo giả thiết quy nạp ta có | EH | [ 12 (n  2)2 ] Do  (n  2)2  n2 | E |   n        n2   4 Do | E | nguyên dương nên ta có | E |  Bài tốn 9: (Rumani TST 2008) Một nhóm người gọi n- cân thoả mãn hai điều kiện: i) Trong số ba người tuỳ ý, tồn hai người quen ii) Trong n người bất kì, ln có hai người khơng quen Chứng minh có nhiều (n  1)(n  2) người nhóm n- cân Phân tích lời giải Từ nhóm người n-cân ta xây dựng đồ thị G  (V ; E) người nhóm n- cân đỉnh, hai người quen biết ta có cạnh đồ thị Ta chứng minh | V | (n  1)(n  2) phương pháp quy nạp theo n Khi n  khẳng định hiển nhiên Giả sử kết cho nhóm (n-1)- cân Xét nhóm n-cân, ta xây dựng đồ thị Lấy A V , với X , Y  M ( A) theo điều kiện i) phải có cạnh nối hai ba đỉnh (A;X;Y) ( X ; Y )  E Điều với X , Y  M ( A) nên theo điều kiện ii) ta suy | M ( A) | n 1 Ta xét trường hợp: Trường hợp 1: | N ( A) | n 1 | V | 2n   (n  1)(n  2) Trường hơp 2: | N ( A) | n Xét tập gồm n  đỉnh N ( A) , thêm vào đỉnh A theo ii) tồn X , Y cho ( X ; Y )  E Rõ ràng X , Y thuộc tập gồm n  đỉnh ban đầu Do N ( A) nhóm (n-1) – cân Theo giả thiết quy nạp ta có | N ( A) | (n  2)(n  1) Suy | V |  n   (n  2)(n  1) (n  1)(n  2)  2 Ta suy kết toán Bài tập tự luyện Bài tập 1: (MOSP, 2011) Xét số thực x1; x2 ; ; xn Chứng minh có khơng q  n2    cặp (i; j ) với  i  j  n cho | xi  x j |   Bài tập 2: Nếu G đồ thị đơn có n đỉnh khơng chứa tứ giác Chứng minh số cạnh G không vượt 13 n(1  4n  3) Bài tập 3: (USA TST 1995) Một đồ thị G gồm n đỉnh k cạnh, G không chứa tam giác Chứng minh chọn đỉnh cho loại bỏ đỉnh cạnh kề ta thu đồ thị cảm sinh có nhiều k (1  4k ) cạnh n2 Bài tập 4: Cho n điểm P1; P2 ; : Pn cho khơng có ba điểm thẳng hàng Người ta kẻ số đoạn thẳng, đoạn nối điểm Biết điểm bất kì, ln có nhât tam giác kẻ Hỏi có đoạn thẳng kẻ Bài tập 5: (Ba Lan 1997) Chứng minh n điểm nằm đường tròn n2 đơn vị có nhiều đoạn thẳng nối cặp điểm chúng có độ dài lớn Bài tập 6: Chứng minh đồ G có 2n+1 đỉnh n2  n  cạnh G chứa tam giác Bài tập 7: (USA TST 2008) Với cặp điểm A( x1; y1 ) B( x2 ; y2 ) mặt phẳng (Oxy), ta đặt d ( A; B) | x1  x2 |  | y1  y2 | Ta gọi cặp ( A; B) (không phân biệt thứ tự) cặp điểm điều hoà  d ( A; B)  Xác định số cặp điểm điều hoà lớn số 100 điểm cho trước mặt phẳng Bài tập 8: Chứng minh đồ thị G gồm n đỉnh k cạnh chứa k 3n (4k  n ) tam giác Bài tập 9: (VMO 2017) Cho số nguyên n  Bảng ô vuông ABCD có kích thước n  n gồm n vuông đơn vị, ô vuông đơn vị tô ba màu: đen, trắng, xám Một cách tô màu gọi đối xứng ô có tâm đường chéo AC tơ màu xám cặp ô đối xứng qua AC tô màu đen màu trắng Người ta điền vào ô xám số 0, ô trắng số nguyên dương ô đen số nguyên âm Một cách điền số gọi k-cân đối (với k nguyên dương) thoả mãn hai điều kiện sau: i) Mỗi cặp ô đối xứng qua AC điền số nguyên thuộc [  k ; k ] ii) Nếu hàng cột giao đen tập số nguyên dương điền hàng tập số ngun dương điền cột khơng giao nhau; hàng cột giao trắng tập số ngun âm điền hàng tập số nguyên âm điền cột khơng giao a) Với n  , tìm giá trị nhỏ k để tồn cách điền k- cân đối cho cách tơ màu đối xứng hình bên b) Với n  2017 , tìm giá trị nhỏ k để với cách tô màu đối xứng, tồn cách điền số k-cân đối 14 TÀI LIỆU THAM KHẢO [1] Lời giải bình luận đề thi VMO 2017, Trần Nam Dũng – Võ Quốc Bá Cẩn – Trần Quang Hùng – Lê Phúc Lữ - Nguyễn Văn Huyện [2] Extremal graph theory, David Conlon [3] Bài giảng tổ hợp dành cho giáo viên 2013, Phan Thị Hà Dương – Viện toán học 15 ...  g (0) 2  (n!)2  điều khơng thể g(0) số nguyên Bài 2: Cho đa thức P( x)  an xn  an 1xn1   a1 x  a0( an  0; n 2) có hệ số số nguyên Chứng minh tồn vô số số nguyên k cho P(x)+k không... a1x  p bất khả quy  x  Trước hết ta nhận xét : Nếu  nghiệm phức Q(x)   Vậy   p  an n   a1  an   a1 mâu thuẩn với cách chọn p Giả sử Q(x) khả quy Tức Q(x)=g(x).h(x) với g(x), h(x)... [2] Nguyễn Văn Mậu Trần Nam Dũng ,Số phức áp dụng [3] Tạp chí Tốn học tuổi trẻ [4] Nguyễn Văn Khuê Lê Mậu Hải,Hàm biến phức Hội An, Ngày 18/6/2017 PHƯƠNG PHÁP ĐẾM SỐ LẦN XUẤT HIỆN CỦA MỖI PHẦN TỬ

Ngày đăng: 03/05/2018, 05:29

TỪ KHÓA LIÊN QUAN

w